GEOMETRY PLEASE HELP ‼️

GEOMETRY PLEASE HELP

Answers

Answer 1

The probabilities are given as follows:

a) Square: 1/6.

b) Not the triangle: 43/48.

How to calculate a probability?

A probability is calculated as the division of the desired number of outcomes by the total number of outcomes in the context of a problem/experiment.

The total area of the figure is given as follows:

12 x 8 = 96 units². (rectangle).

The area of the square is given as follows:

4² = 16 units² (square of the side lengths).

Hence the probability of the square is given as follows:

p = 16/96

p = 1/6.

The area of the triangle is given as follows:

A = 0.5 x 4 x 5 = 10 units². (half the multiplication of the side lengths).

Hence the complement of the area of the triangle is of:

96 - 10 = 86 units².

And the probability of the complement is of:

86/96 = 43/48.

More can be learned about probability at https://brainly.com/question/24756209

#SPJ1


Related Questions

Which statement is true about all perfect cubes?
A
A perfect cube represents 3 times the area of a face of the cube.
B
A perfect cube represents the sum of 9 edge lengths of the cube.
A perfect cube represents the volume of a cube with equal integer side lengths.
D
A perfect cube represents the surface area of a cube with equal integer side lengths.
C

Answers

The answer he c have a good day

If nori made 2% in interest on $5,000 and her brother Sean made 1% in interest on $10,000, who made more money in interest?

Answers

Answer: Nori

Step-by-step explanation:

2% of 5000 = 100

1% of 1000 = 10

Under which
transformation would AA'B'C', the wen 2.
image of AABC, not be congruent to AABC?
a. reflection over the y-axis
b.
rotation of 90° clockwise about the origin
c. translation of 3 units right and 2 units down
d. dilation with a scale factor of 2 centered at
the origin

Answers

The answer is a reflection over the y-axis

rotation 90 degrees clockwise about the origin, ignore the dots i kinda started it then i got lost

Answers

When the points are rotated 90 degrees clockwise about the origin, the result is:

I: (1, -3)J: (-1, -5)H: (-3, -3)

How to rotate about the origin ?

To rotate a point 90 degrees clockwise about the origin, you can use the following rule: (x, y) becomes (y, -x). Let's apply this rule to the given points:

I - (3, 1)

Rotated I: (1, -3)

J - (5, -1)

Rotated J: (-1, -5)

H - (3, -3)

Rotated H: (-3, -3)

So, after a 90-degree clockwise rotation about the origin, the new coordinates of the points are:

I: (1, -3)

J: (-1, -5)

H: (-3, -3)

Find out more on rotation at https://brainly.com/question/30316631

#SPJ1

i need help fast!!!!

Answers

Answer:

1st choice:  1/4(y - 10) = 2/3

Step-by-step explanation:

the "variable" is y

"is" means "=" (equals sign)

one fourth =  1/4

"difference of" means subtract

Answer:  1/4(y - 10) = 2/3

Find y such that
∫x^5 dx = ∫ x^y dx

Answers

The value of y that satisfies the equation [tex]\int x^5 dx = \int x^y dx[/tex] is y = -1.

We know that the indefinite integral of x^5 dx is (1/6) x^6 + C, where C is

the constant of integration. Therefore:

[tex]\int x^5 dx = (1/6) x^6 + C[/tex]

We want to find y such that [tex]\int x^5 dx = \int x^y dx[/tex]. Using the power rule of integration, the indefinite integral of [tex]x^y[/tex] dx is [tex](1/(y+1)) x^{(y+1)} + C[/tex], where C is the constant of integration. Therefore:

[tex]\int x^y dx = (1/(y+1)) x^{(y+1)} + C[/tex]

For these two integrals to be equal, we need:

[tex](1/6) x^6 + C = (1/(y+1)) x^{(y+1) } + C[/tex]

Subtracting C from both sides, we get:

[tex](1/6) x^6 = (1/(y+1)) x^{(y+1)}[/tex]

Multiplying both sides by (y+1), we get:

[tex](1/6) x^6 (y+1) = x^{(y+1)}[/tex]

Now, we can equate the powers of x on both sides:

[tex]x^6 (y+1) = x^{(y+1)}[/tex]

Using the fact that[tex]x^a \times x^b = x^{(a+b)}[/tex], we can simplify the left-hand side:

[tex]x^(6(y+1)) = x^{(y+1)}[/tex]

Now, we can equate the exponents on both sides:

6(y+1) = y+1

Simplifying, we get:

6y + 6 = y + 1

5y = -5

y = -1

for such more question on integral

https://brainly.com/question/22008756

#SPJ11

It is kinda hard but just try it

Answers

Answer:

we 1st can get the weight of rat by

1 rat and 1 cat + 1 dog and rat = 30

2 rat + 1 cat + 1 dog = 30

Then 1 rat and cat measure 24 so

2 rat + 24 =30

2 rat + 24 =30 1 rat = 3 kg

2 rat + 24 =30 1 rat = 3 kg 1 cat + 1 rat = 10

2 rat + 24 =30 1 rat = 3 kg 1 cat + 1 rat = 101 cat + 3kg = 10

2 rat + 24 =30 1 rat = 3 kg 1 cat + 1 rat = 101 cat + 3kg = 101 cat = 7kg and

2 rat + 24 =30 1 rat = 3 kg 1 cat + 1 rat = 101 cat + 3kg = 101 cat = 7kg and 1 dog + 1 rat = 20 kg

2 rat + 24 =30 1 rat = 3 kg 1 cat + 1 rat = 101 cat + 3kg = 101 cat = 7kg and 1 dog + 1 rat = 20 kg 1 dog + 3kg = 20 kg

2 rat + 24 =30 1 rat = 3 kg 1 cat + 1 rat = 101 cat + 3kg = 101 cat = 7kg and 1 dog + 1 rat = 20 kg 1 dog + 3kg = 20 kg 1 dog = 17kg

so we get the weight of each now we r going to sum them 1 rat + 1 cat + 1 dog = x

1 rat + 1 cat + 1 dog = x 3 kg + 7 kg + 17 kg = x

1 rat + 1 cat + 1 dog = x 3 kg + 7 kg + 17 kg = x 27 kg = x ..... is the mass of 3 of them

Rat=3kg cat=7kg dog=17kg all three combined = 27kg

Out of all the people who like chocolate, what is the relative frequency for selecting a teen?

Answers

The relative frequency for selecting a teen out of all the people who like chocolate is calculated by dividing the number of teens who like chocolate (N) by the total number of people who like chocolate (T).

To find the relative frequency for selecting a teen out of all the people who like chocolate, you need to follow these steps:

Step 1: Determine the total number of people who like chocolate (let's call this T).
Step 2: Determine the number of teens who like chocolate (let's call this N).
Step 3: Calculate the relative frequency by dividing the number of teens who like chocolate (N) by the total number of people who like chocolate (T).

To learn more about relative frequency

https://brainly.com/question/18834138

#SPJ11

determine if each of the numbers below is a solution to the inequality 3x-2<2-2x

Answers

The solution set of the inequality 3x-2 < 2-2x is:

(4/5, ∞)

Which numbers are solutions for the inequality?

To find this we need to isolate the variable in the inequality.

Here we have:

3x - 2 < 2 - 2x

add 2x in both sides and add 2 in both sides, then we will get:

3x + 2x < 2 + 2

5x < 4

Now we can divide both sides by 5 to get:

x < 4/5

That is the inequality solved.

Then the solution set of the inequality is:

(4/5, ∞)

The set of all real numbers larger than 4/5.

Learn more about inequalities:

https://brainly.com/question/24372553

#SPJ1

the top of the farm silo is a hemisphere with a radius of 9ft. the bottom of the silo is a cylinder with a height of 35ft. how many cubic feet of grain can the solo hold? use 3.14 for pi and round your answer to the nearest cubic foot.​

Answers

To find the total volume of the silo, we need to add the volume of the hemisphere on top to the volume of the cylinder at the bottom.

The volume of a hemisphere is given by:

V_hemi = (2/3)πr^3

where r is the radius of the hemisphere.

Substituting r = 9ft, we get:

V_hemi = (2/3)π(9ft)^3

= 1521π ft^3

The volume of a cylinder is given by:

V_cyl = πr^2h

where r is the radius of the cylinder and h is its height.

Substituting r = 9ft and h = 35ft, we get:

V_cyl = π(9ft)^2(35ft)

= 2673π ft^3

Therefore, the total volume of the silo is:

V_silo = V_hemi + V_cyl

= 1521π + 2673π

= 4194π ft^3

≈ 13160 ft^3

Rounding to the nearest cubic foot, the silo can hold approximately 13160 cubic feet of grain.

consider light falling on a single slit, of width 1.2 μm, that produces its first minimum at an angle of 32.3°. randomized variables θ = 32.3° w = 1.2 μm

Answers

The wavelength of the light is approximately 0.687 μm.

Using the single slit diffraction formula, we have:

sin θ = (mλ) / w

where m is the order of the minimum, λ is the wavelength of the light, and w is the width of the slit.

We can rearrange the formula to solve for the wavelength of the light:

λ = (w sin θ) / m

Plugging in the given values, we get:

λ = (1.2 μm)(sin 32.3°) / 1 = 0.687 μm

Therefore, the wavelength of the light is approximately 0.687 μm.

The wavelength is the distance between two consecutive peaks or troughs in a wave. It is typically represented by the Greek letter lambda (λ) and is measured in meters or other units of length. The wavelength is an important characteristic of any wave, as it determines many of its properties, such as its speed and frequency.

Learn more about wavelength

https://brainly.com/question/31143857

#SPJ4

Find the derivative of the vector function r(t) = ln(7-t^2)i + sqrt(13+tj – 4e^{9t} r’(t) =

Answers

The derivative of the vector function is: r'(t) = (-2t/(7-t^2)) i + (1/(2sqrt(13+t))) j - 36e^(9t) k

We are given a vector function r(t) = ln(7-t^2)i + sqrt(13+t)j – 4e^(9t)k, and we need to find its derivative r'(t).

The derivative of a vector function is obtained by differentiating each component of the vector function separately.

So, let's differentiate each component:

r(t) = ln(7-t^2)i + sqrt(13+t)j – 4e^(9t)k

r'(t) = (d/dt) ln(7-t^2) i + (d/dt) sqrt(13+t) j - (d/dt) 4e^(9t) k

Using the chain rule of differentiation, we have:

r'(t) = -2t/(7-t^2) i + 1/(2sqrt(13+t)) j - 36e^(9t) k

Therefore, the derivative of the vector function is:

r'(t) = (-2t/(7-t^2)) i + (1/(2sqrt(13+t))) j - 36e^(9t) k

To learn more about vector function visit: https://brainly.com/question/8005711

#SPJ11

A function is a rule that assingns each value of independent variable to exactly value of the dependent variable

Answers

A function is a rule that assingns each value of independent variable to exactly one value of the dependent variable.

A function is a mathematical concept that relates two sets of values, known as the domain and the range. The domain is the set of independent variables, while the range is the set of dependent variables. A function is a rule that assigns to each value in the domain exactly one value in the range.

For example, if we have a function f(x) = 2x + 3, the domain would be any possible value of x, and the range would be any possible value of 2x + 3. So if we put x = 2, then f(x) = 2(2) + 3 = 7. Therefore, the function assigns the value of 7 to the value of 2 in the domain.

Functions are used in various branches of mathematics, science, and engineering to model and analyze relationships between two or more variables. They are an important concept in calculus, where they are used to study rates of change and optimization problems.

Learn more about function at https://brainly.com/question/21084589

#SPJ11

An 8-sided solid is labeled with faces 1, 2, 3, skip ,4, 5, 6, skip. what is the sample space for the number solid, and what is the probability of rolling a 1?

Answers

The sample space for the number solid is {1, 2, 3, 4, 5, 6} and the probability of rolling 1 is 1/6.

The sample space refers to the set of all possible outcomes of an experiment, while a sample value is a specific outcome in the sample space.

For the given 8-sided solid, the sample space would be {1, 2, 3, 4, 5, 6}, as the faces labeled "skip" are not counted as sample values.

Now, let's calculate the probability of rolling a 1. Probability is the likelihood of a particular outcome occurring, which can be calculated by dividing the number of successful outcomes (in this case, rolling a 1) by the total number of possible outcomes.

The total number of possible outcomes is 6 (1, 2, 3, 4, 5, and 6). There is only one successful outcome: rolling a 1.

So, the probability of rolling a 1 is:

P(1) = (Number of successful outcomes) / (Total number of possible outcomes)
P(1) = 1 / 6

Thus, the probability of rolling a 1 on this 8-sided solid is 1/6 or approximately 0.1667, or 16.67%.\

To learn more about sample space: https://brainly.com/question/2117233

#SPJ11

a police car is parked 40 feet due north of a stop sign on straight road. a red car is travelling towards the stop sign from a point 160 feet due east on the road. the police radar reads that the distance between the police car and the red car is decreasing at a rate of 100 feet per second. how fast is the red car actually traveling along the road?

Answers

The red car is actually traveling along the road at a speed of approximately 26.67 feet per second.

We can start by drawing a diagram of the situation:

     P (police car)

      |

      |

      |

40    |    S (stop sign)

-------|--------------------

      | 160

      |    R (red car)

Let's use the Pythagorean theorem to find the distance between the police car and the red car at any time t:

d(t)² = 40² + (160 - v*t)²

Where v is the speed of the red car in feet per second, and d(t) is the distance between the police car and the red car at time t.

We want to find how fast the red car is actually traveling along the road, so we need to find v when the distance between the police car and the red car is decreasing at a rate of 100 feet per second:

d'(t) = -100

We can take the derivative of the equation for d(t) with respect to time:

2d(t)d'(t) = 0 + 2(160 - v*t)(-v)

Simplifying and plugging in d'(t) = -100, we get:

-4000 + 2v²t = -100(160 - vt)

Solving for v, we get:

v = 80/3 ≈ 26.67 feet per second

Learn more about Pythagorean theorem

https://brainly.com/question/14930619

#SPJ4

Line x is parallel to line y. Line z intersect lines x and y. Determine whether each statement is Always True.

Answers

Line x is perpendicular to line y. Line z crosses lines x and y. Only statements 3 and 4 are true.

∠6 = ∠8 is not true because they both lie on the same plane and makes an angle of 180° and can never be true. ∠6 = ∠1 is also not true because ∠1 is clearly obtuse angle and ∠6 is clearly acute angle so they cannot be equal. Hence, statement a and b are false.

∠7 = ∠3 is always true because they are corresponding angles and corresponding angles are always equal. m∠2 + m∠4 = 180° is also true because they lie on same plane and have common vertex and hence, they are supplementary angles and make a sum of 180°.  Hence, statement 3 and 4 is always true.

To know more about corresponding angles:

https://brainly.com/question/12521848

#SPJ1

The mainsail of a boat has the dimensions shown. If the mainsail is a right triangle, what is the exact height of the mainsail shown?

a.) 2√6 feet
b.) 24 feet
c.) 4√78 feet
d.) 2√410 feet

Answers

Step-by-step explanation:

use Pythagorean theorem to find the height

c = 38 ft

a = 14 ft

a² + b² = c²

(14)² + b² = (38)²

b² = 1444 - 196

b² = 1248

b = √1248

b = √16 × 78

b = 4√78 feet

#CMIIW

Francium is a radioactive element discovered by Marguerite Perey in 1939 and named after her country. Francium has a half-life of 22 minutes.
a) Write an exponential function that models the mass how many grams remain from a 480-gram sample after t minutes.
b) How many grams remain after 2 hours?

Answers

After 2 hours, approximately 4.38 grams of Francium remain from the 480-gram sample.

What is Algebraic expression ?

An algebraic expression is a combination of variables, constants, and mathematical operations such as addition, subtraction, multiplication, and division. It may contain one or more terms, with each term separated by a plus or minus sign. Algebraic expressions are used in algebra to represent mathematical relationships and formulas.

a) To write an exponential function that models the mass of Francium remaining after t minutes, we can use the formula:

N = N0 * [tex](1/2)^{(t / t1/2)}[/tex]

where N is the amount remaining after time t, N0 is the initial amount, t1/2 is the half-life, and (t/t1/2) means raised to the power of t/t1/2.

In this case, the initial amount is 480 grams, the half-life is 22 minutes, and we want to find the amount remaining after t minutes. Therefore, the exponential function that models the mass of Francium remaining after t minutes is:

N = 480 * [tex](1/2)^{t/22}[/tex]

b) 2 hours is equal to 120 minutes. To find how many grams of Francium remain after 2 hours, we can substitute t = 120 into the exponential function we found in part a):

N = 480 *[tex](1/2)^{ (120 / 22) }[/tex] ≈ 4.38 grams

Therefore, after 2 hours, approximately 4.38 grams of Francium remain from the 480-gram sample.

To learn more about Algebraic expression from given link.

brainly.com/question/31238826

#SPJ1

Find the critical points for the function f(x, y) = x³ + y³ – 9x² – 3y - 6 = and classify each as a local maximum, local minimum, saddle point, or none of these. critical points: (give your points as a comma separated list of (x,y) coordinates.) classifications: (give your answers in a comma separated list, specifying maximum, minimum, saddle point, or none for each, in the same order as you entered your critical points)

Answers

The critical points and their classifications are:

(0, 1) - saddle point

(0, -1) - saddle point

(6, 1) - local minimum

(6, -1) - local minimum

To find the critical points of the function f(x, y) = x³ + y³ – 9x² – 3y - 6, we need to find the points where the partial derivatives of f with respect to x and y are zero.

∂f/∂x = 3x² - 18x = 3x(x - 6)

∂f/∂y = 3y² - 3 = 3(y² - 1)

Setting these partial derivatives equal to zero and solving for x and y, we get:

x = 0 or x = 6

y = ±1

So the critical points are (0, 1), (0, -1), (6, 1), and (6, -1).

To classify each critical point, we need to compute the second partial derivatives of f:

∂²f/∂x² = 6x - 18

∂²f/∂y² = 6y

∂²f/∂x∂y = 0

At (0, 1):

∂²f/∂x² = -18 < 0 (concave down)

∂²f/∂y² = 6 > 0 (concave up)

So (0, 1) is a saddle point.

At (0, -1):

∂²f/∂x² = -18 < 0 (concave down)

∂²f/∂y² = 6 > 0 (concave up)

So (0, -1) is a saddle point.

At (6, 1):

∂²f/∂x² = 18 > 0 (concave up)

∂²f/∂y² = 6 > 0 (concave up)

So (6, 1) is a local minimum.

At (6, -1):

∂²f/∂x² = 18 > 0 (concave up)

∂²f/∂y² = 6 > 0 (concave up)

So (6, -1) is a local minimum.

For more such questions on Critical points.

https://brainly.com/question/30889798#

#SPJ11

There are 30 skittles in a box, for every 5 green there are 7 yellow, how many yellows are there in the box

Answers

There are 42 yellow skittles in the box.

Based on the given information, we know that the ratio of green skittles to yellow skittles is 5:7. This means that for every 5 green skittles, there are 7 yellow skittles.

To find out how many yellow skittles are in the box, we need to know how many sets of 5 green skittles there are. We can do this by dividing the total number of skittles in the box (30) by 5 (since there are 5 green skittles for every set).
30 ÷ 5 = 6

This means there are 6 sets of 5 green skittles in the box.

Now we can use the ratio of 5:7 to find out how many yellow skittles there are in each set:
5 green skittles : 7 yellow skittles

Since there are 7 yellow skittles in each set, we can find the total number of yellow skittles by multiplying 7 by the number of sets (6):
7 x 6 = 42

There are 42 yellow skittles in the box.

To know more about "Ratio" refer here:

https://brainly.com/question/17223098#

#SPJ11

please do both will give brainliest and it's for 72 points

Answers

Step-by-step explanation:

Pick any of the two points...I'll use the first two

calculate slope:     m  = ( y1-y2) / (x1-x2) =  (-14 - -5) / (-2 -1) = -9/-3 = 3

   equation of a line in slope intercept form is y = mx+ b

      so now you have    y = 3x + b  

             sub in any of the x,y  points given (8,16)  to calculate 'b'

                             16 = 3 (8) + b

                                  b = -8

so your first line is     y = 3x - 8

In a similar fashion, for the second one   m = - 5/8    and b = 2

           y = -5/8 x + 2

x-3 5. The function f(x)=- has X? - 8x+15 Math 1P97 Final Exam April 2010 page 3 of 19 a. a discontinuity at x = 3 only b. discontinuities at = 3 and x = 5 c. no discontinuities d. a discontinuity at x = 5 only e, none of the above

Answers

The correct option is: (d) a discontinuity at x = 5 only.

How to find which function f(x)=- has X?

The function f(x) is defined as:

[tex]f(x) = (x-3)/(x^2 - 8x + 15)[/tex]

The denominator of this function can be factored as:

[tex]x^2 - 8x + 15 = (x - 3)(x - 5)[/tex]

So the function can be rewritten as:

f(x) = (x - 3)/[(x - 3)(x - 5)]

Simplifying this expression, we get:

f(x) = 1/(x - 5)

Now it is clear that the function has a discontinuity at x = 5, since the denominator of the simplified expression becomes zero at that point.

Therefore, the correct option is:

d. a discontinuity at x = 5 only

Learn more about discontinuities in functions.

brainly.com/question/28783981

#SPJ11

4 Il y f(x, y) da = Sot Shot Sot Staf (x, y) dxdy x D
Characteristics of the drawing of D, you can choose several answers:
1. It is the region in the first quadrant that is bounded from the right by the line x = 2
2. It is the region in the first quadrant that is bounded above by y = x
3. It is the region in the first quadrant that is bounded from the left by the line x = 0
4. It is the region in the first quadrant that is bounded above by y = x2
5. It is the region in the first quadrant that is bounded below by y = 0
6. It is the region in the first quadrant that is bounded below by y = 2
which of these 6 options is correct?

Answers

The correct option is option 3.

How to determine the boundaries of the region?

Based on the given integral, region D is in the first quadrant, and its boundaries are not explicitly given. However, we can deduce the boundaries of D by looking at the integrand. Since the integrand is f(x,y), we can see that we are integrating over the entire region D, which means that D must be the rectangle that contains all the other regions mentioned in the options.

Therefore, option 1 is not correct, as D is not bounded from the right by x=2, but rather extends indefinitely to the right. Option 2 is also not correct, as D extends beyond the line y=x. Option 4 is not correct either, as D is not bounded above by y=x^2, but rather extends beyond it. Options 5 and 6 are also not correct, as D extends beyond the lines y=0 and y=2.

Therefore, the correct option is option 3, which states that D is the region in the first quadrant that is bounded from the left by the line x=0. This is correct, as D extends indefinitely to the right, and is bounded from the left by x=0.

Learn more about the boundaries of the region.

brainly.com/question/29674302

#SPJ11

3) Find the maximum and minimum values of f(x,y) = xyon the region inside the triangle whose vertices are (6,2), (0,3), and (6.0).

Answers

Therefore, the maximum value of f(x,y) inside the triangle is 80/9, which occurs along the line y = (-1/2)x + 4 at the point (8/3, 10/3), and the minimum value is -32, which occurs at the critical point (-8,4).

To find the maximum and minimum values of f(x,y) = xy on the region inside the triangle whose vertices are (6,2), (0,3), and (6,0), we use the method of Lagrange multipliers.

First, we need to find the critical points of f(x,y) subject to the constraint that (x,y) lies inside the triangle. We can express this constraint using the equations of the lines that form the sides of the triangle:

y = (-1/2)x + 4

y = (3/2)x

y = 0

Next, we set up the Lagrange multiplier equation:

∇f = λ∇g

where g(x,y) is the equation of the constraint, i.e., the triangle.

We have:

f(x,y) = xy

∇f = <y, x>

g(x,y) = y - (-1/2)x - 4 = 0

∇g = <-1/2, 1>

Setting ∇f = λ∇g, we get:

y = (-1/2)λ

x = λ

Substituting these into the constraint equation, we get:

(-1/2)λ - 4 = 0

Solving for λ, we get:

λ = -8

Substituting this into y = (-1/2)λ and x = λ, we get:

x = -8 and y = 4

Therefore, the only critical point of f(x,y) inside the triangle is (-8,4).

Next, we need to check the values of f(x,y) at the vertices and along the sides of the triangle.

At the vertices:

f(6,2) = 12

f(0,3) = 0

f(6,0) = 0

Along the line y = (3/2)x:

f(x, (3/2)x) = (3/2)x^2

Using the vertex (6,2) and the x-intercept (4/3, 2), we can see that the maximum value of (3/2)x^2 on this line occurs at x = 4. Therefore, the maximum value of f(x,y) along this line is:

f(4,6) = 24

Along the line y = (-1/2)x + 4:

f(x, (-1/2)x + 4) = (-1/2)x^2 + 4x

Using the vertex (6,2) and the x-intercept (8,0), we can see that the maximum value of (-1/2)x^2 + 4x on this line occurs at x = 8/3. Therefore, the maximum value of f(x,y) along this line is:

f(8/3,10/3) = 80/9

Finally, we need to check the values of f(x,y) at the critical point (-8,4). We have:

f(-8,4) = -32

For more similar questions on minimum function

brainly.com/question/10439235

#SPJ11

What is a good percentage (in decimal form) to multiply your earning to estimate your paycheck?

Answers

To estimate your paycheck, a good percentage to multiply your earning by would be 0.75 or 75%. When calculating your paycheck, it's important to account for taxes, deductions, and other withholdings that may be taken out of your gross pay.  

This accounts for taxes, deductions, and other withholdings that are typically taken out of your paycheck before you receive your net pay. For example, if you earn $1,000 per pay period, multiplying by 0.75 would give you an estimated net pay of $750. However, keep in mind that this is just an estimate and your actual net pay may vary depending on your specific tax situation and other factors.

Learn more about paycheck,

https://brainly.com/question/31171390

#SPJ4

Tanya made this graph that represents the total cost for each of the three locations. Depending on the number of students that attend. Which function represents the cost of the restaurant 

Answers

The functions that represents the cost are

(a) y = 8800, (b) y = 1900 + 4/7x and (c) y = 4800, x ≤ 150;  y = 1200 + 24x x > 150

Identifying the function that represents the cost

From the question, we have the following parameters that can be used in our computation:

The graph


The function (a) is a horizontal line that passes through y = 8800

So, the function is

y = 8800

The function (b) is a linear function that passes through

(0, 1900) and (175, 2000)

So, the function is

y = 1900 + 4/7x

The function c is a piecewise function with the following properties

Horizontal line of y = 4800 uptill x = 150Linear function of (150, 4800) and (200, 6000)

So, the function is

y = 4800, x ≤ 150

y = 1200 + 24x x > 150

Read more about functions at

https://brainly.com/question/27915724

#SPJ1

Which point on the number line has the least absolute value?​

Answers

The point with the least absolute value on the number line is always the point zero.

The absolute value of a number is the distance that number is from zero on the number line. Therefore, the point on the number line with the least absolute value is the point closest to zero. This point is located at zero itself, as it is the point on the number line that is equidistant from both the positive and negative numbers.

To further explain, consider the following examples:

- The point 3 is 3 units away from zero, but the point -3 is also 3 units away from zero.
- The point 5 is 5 units away from zero, but the point -5 is also 5 units away from zero.
- The point 0 is 0 units away from zero, making it the point with the least absolute value on the number line.

In conclusion, the point with the least absolute value on the number line is always the point zero.

To know more about absolute value, visit:

https://brainly.com/question/1301718#

#SPJ11

240:360=?:120 (Please quickly)​

Answers

Answer:

? equals 80

Step-by-step explanation:

Un terreno de forma rectangular tiene un perímetro de 105 metros. Si el ancho es la mitad del largo, ¿Cuáles son las medidas del terreno? *

Answers

Sea "l" la medida del largo del terreno y "a" la medida del ancho del terreno.

De acuerdo con el problema, el ancho es la mitad del largo, es decir, a = l/2.

El perímetro de un rectángulo se calcula sumando las longitudes de sus cuatro lados, por lo que en este caso:

Perímetro = 2l + 2a = 2l + 2(l/2) = 3l

Sabemos que el perímetro es de 105 metros, entonces:

3l = 105

l = 105/3 = 35

Por lo tanto, el largo del terreno es 35 metros. Y, como el ancho es la mitad del largo, entonces:

a = l/2 = 35/2 = 17.5

Por lo tanto, el ancho del terreno es de 17.5 metros.

En resumen, las medidas del terreno son 35 metros de largo y 17.5 metros de ancho.

To know more about del terreno refer here:

https://brainly.com/question/31136130

#SPJ11

) Nadia buys 4 1/5 pounds of plums. Nadia used a 55 cent coupon off her entire purchase. Her total after the coupon was $3. 23. If c represents the cost per pound for the plums, create and solve an equation to determine the cost per pound for the plums

Answers

If c represents the cost per pound for the plums, the cost per pound for the plums is $0.90.

First, we need to determine the total cost of the plums before the coupon was applied.

4 1/5 pounds can be written as a mixed number:

4 1/5 = 21/5

So, the total cost of the plums without the coupon can be found by multiplying the cost per pound (c) by 21/5:

Total cost = c * 21/5

Now we can create an equation to represent the total cost after the coupon was applied:

Total cost - coupon = $3.23

Substituting the expression for total cost:

c * 21/5 - 0.55 = 3.23

To solve for c, we can start by adding 0.55 to both sides:

c * 21/5 = 3.78

Then, we can isolate c by multiplying both sides by the reciprocal of 21/5:

c = 3.78 / (21/5)

c = 0.90

Therefore, the cost per pound for the plums is $0.90.

More on cost: https://brainly.com/question/13001095

#SPJ11

Other Questions
What does sterling brown mean when he mentions the chain gang?chain gang nevahhunh let me go; chain gang nevahhunh let me go; po' los' boy, bebby, evahmo' . . .a. brown is referring to people in general, who are chained in their mindsb. brown is referring to gangs in ghetto chained to their crimes and povertyc. brown is referring to men perhaps prisoners singing/chanting while workingd. brown is referring to slaves in chain Place yourself in the position of the son, his father, and the master in this agreement Explain the advantages and disadvantages to each under the terms of the agreement. for about 2 decades 10 of electricity came from what source Cuando voy a la playa me encanta nadar en el aguaO aObOcOddel marde la arenade la piscinadel lago(1 point 3. Use the definition of the Riemann Sum to evaluate S12(x3 2x)dx. + - 4. Evaluate: S2x2+x=2 5. Given the velocity in meters/second for v(t) = 8 2t, 1 st 56 a.) find the displacement of the particle over the given time interval; b.) find the distance traveled by the particle over the given time interval. : How does the donkey respond to the challenge?A He disquises himself as a tiger.B He runs away to a farmer's pasture.refuses to C He didnt carry any more loads.D He lets other animals think he is a tiger. In the united states, the number of children under age 18 per family is skewed right with a mean of 1.9 children and a standard deviation of 1.1 children.analyst 1 wants to calculate the probability that a randomly selected family from the united states has at least 2 children.analyst 2 wants to calculate the probability that if 40 families from the united states are randomly selected, the mean number of children per family is at least 2 children.what sample size does analyst 1 plan to use?enter an integer. what sample size does analyst 2 plan to use?enter an integer. 1. Write a JavaScript statement or a set of statements to accomplish each of the following tasks: a) Sum the odd integers between 1 and 99. Use a for structure. Assume that the variables sum and count have been declared. B) Calculate the value of 2. 5 raised to the power of 3. Use the pow method. C) Print the integers from 1 to 20 by using a while loop and the counter variable x. Assume that the variable x has been declared, but not initialized. Print only five integers per line. D) Repeat Exercise (c), but using a for statement. G Sharla wanted to know how many minutes per hour a radio station typically plays music. She collected the following data fromstations,Radio Station Music3630 31 32 33 34 35Minutes per HourBy how many minutes would her median time change if she added another radio station playing 37 minutes?O A. 0. 45OB. 0. 5OC. the median did not changeOD. 0. 4 Select the correct equation that can be used to represent the lumens, L, after x screen layers are added. A. L = 750(0. 975)x B. L = 750(1. 25)x C. L = 750(0. 25)x D. L = 750(0. 75)x From the presentation, choose an example of an environmental justice issue that impacts workers or communities. Explain what the issue is, and how modern agriculture is contributing to it. A friend of yours starts his own business. He would like to expand his client base to include the government, but he believes his small business would be ignored. Based on your knowledge from the text, you tell him Group of answer choices that any government, federal, state, or local would laugh at the size of his business. Although the government will deal with small businesses, he will never make a profit off a government contract. The government buys products from all sizes of business, but there is some red tape. That he's absolutely right, the government doesn't deal with small businesses. The government rarely considers new suppliers when making purchasing decisions Describe what the weather would be like as a warm front passes. Then, describe what would occur as a cold front passes PLEASE HELP THIS A FRESHMEN QUESTION Town officials want to estimate the number of households that own a dog. Answer the following. There are 300 households in the town. Estimate how many households that own a dog __ households -3 3/7 times 5 5/6 ........ What is a solvent front? Identify the correct verb mood. I wish I were already in college. Indicative conditional subjunctive interrogative An electric field of 2250 N/C is produced by a charge of 4.82 x 10^-11 C. For this field strength, what is the distance to the charge? (Kc = 8.99 x 10^9 NM^2 / C^2 ) If 3/10 of a number is equal to 1/4 what is the number